隨機過程

Ito、隨機指數和 Girsanov

  • July 12, 2015

這是一個由兩部分組成的問題,涉及 Girsanov 中使用的測量密度的變化,其次是隨機指數。

在閱讀有關 Girsanov 的筆記時,據說測量密度鞅的變化可以寫成:

$$ \begin{align} \rho_t = \exp \left[- \int_{0}^{t} \lambda_s , dW_s - \tfrac{1}{2}\int_{0}^{t} \lambda_{s}^{2} , ds \right] \end{align} $$ 據說,使用 Ito 引理可以直接驗證 $ \rho_t $ 是(誰)給的

$$ \begin{align} d\rho_t = -\rho_t , \lambda_t , dW_t \end{align} $$ 我想我已經解決了這個問題並將其應用如下:

$$ \begin{align} \rho_t &= exp\left[-\lambda_t W_t - \tfrac{1}{2} , \lambda_{t}^{2} , t \right]\ d\rho_t &= \frac{\partial\rho_t}{\partial t} dt + \frac{\partial \rho_t}{\partial W} dW_t + \tfrac{1}{2} \frac{\partial^2 \rho_t}{\partial W^{2}} (dW_{t})^2\ &= -\tfrac{1}{2} \lambda_{t}^{2}\exp\left[\dots\right] - \lambda_t\exp\left[\dots\right] dW_{t} + \tfrac{1}{2} \lambda_{t}^2 \exp\left[\dots\right] (dW_{t})^{2}\ &= -\tfrac{1}{2},\lambda_{t}^{2},\rho_{t},dt - \lambda_{t},\rho_{t}dW_{t} + \tfrac{1}{2},\lambda_{t}^{2},\rho_{t} dt\ &= -\lambda_{t}\rho_{t},dW_{t} \end{align} $$ $ \textbf{Question 1} $ - 它是否正確?

隨機指數表示為:

$$ \begin{align} \mathcal{E}t(X) = \exp\left[ X_t - \tfrac{1}{2} \langle X,X \rangle{t} \right] \end{align} $$ $ \textbf{Question 2} $ - 我相信 $ \langle X,X \rangle_t $ 是二次變化,我是否應該以與我相同的方式解釋 $ \int_{0}^{t} \lambda_{t}^{2},ds $ 上面測量密度變化中的術語?

據說(Filipovic - Term-Structure Models)如果 $ X_t $ 是一個連續局部鞅 $ X_0 = 0 $ ,然後使用 Ito 可以看到 $ d\mathcal{E}_t(X) = \mathcal{E}_t(X)dX_t $ .

我的解決方案如下:

$$ \begin{align} d\mathcal{E}t(X) &= \exp\left[X_t - \tfrac{1}{2}\langle X \rangle_t \right]\left(dX_t - \tfrac{1}{2}d\langle X \rangle_t \right) + \tfrac{1}{2}\exp\left[X_t - \tfrac{1}{2}\langle X \rangle_t \right]d\langle X \rangle_t\ &= \exp\left[X_t - \tfrac{1}{2}\langle X \rangle_t \right] dX_t\ &= \mathcal{E}{t}dM_t\ \text{obviously $\mathcal{E}0(X) = 1.$} \end{align} $$ $ \textbf{Question 3} $ - 我很難理解這裡的符號,並且看不到 Ito 在這種情況下是如何應用的(因為我看不到 $ dt $ 和 $ dW $ 條款)。我將不勝感激任何幫助向我展示如何在這種情況下應用 ito(以及為什麼很明顯 $ \mathcal{E}{0}(X) = 1 $ ).

非常感謝,

約翰

對於問題一,身份

$$ \begin{align*} \rho_t = \exp\big(-\lambda_t W_t - \frac{1}{2} \lambda_t^2t\big) \end{align*} $$ 似乎不正確,除非 $ \lambda_t $ 是一個常數。 對於問題二,是的。如果 $ X_t = -\int_0^t \lambda_s dW_s $ , 然後 $ \langle X \rangle_t = \int_0^t \lambda_s^2 ds $ .

對於問題三,你需要注意的是

$$ \begin{align*} \langle X \rangle_t = \int_0^t \frac{\partial\langle X \rangle_s}{\partial s}ds. \end{align*} $$ 然後 $ d\langle X, \langle X \rangle \rangle_t = 0 $ 和 $ d\langle \langle X \rangle, \langle X \rangle \rangle_t = 0 $ .

引用自:https://quant.stackexchange.com/questions/17293